Zwei Aufgaben zu Mathe/Physik



  • Diese Aufgaben sind von der Hochschule Wismar, und zusammen sollte die Bearbeitungszeit nicht länger als 10 Minuten dauern, also 5 Minuten pro Aufgabe.

    a) Berechne x für a>0 und n Element ganze Zahl: 2 ln a - 1/2 ln a/3 = ln 2/x

    b) Fließt ein Gleichstrom durch eine verdünnte Kupfersulfatlösung, so entsteht am negativen Pol metallisches Kupfer. Die abgeschiedene Kupfermenge ist sowohl zur Dauer des Stromflusses als auch zur Stromstärke direkt proportional. Bei einer Stromstärke von 0.4 Ampere werden in 15 Minuten 0.12 g Kupfer abgeschieden. Wie lange dauert es, bis bei einer Stromstärke von 1 Ampere 0.24 g Kupfer abgeschieden werden?

    Zu b) Ich hätte einfach 15 x 2 = 30 Minuten und 30 / (1/0.4) = 30 / 2.5 = 12 Minuten gerechnet, aber so einfach kann es doch nicht sein, oder? Dann hätte ich für b) nur 1 Minute gebraucht.



  • @Fragender
    ad a) Es kommt kein nn in deiner Formel 2 ln a - 1/2 ln a/3 = ln 2/x vor. Ist die Vorgabe nZn \in \mathbb Z nur zur Verwirrung angegeben oder Schreib/Lesefehler?

    ad b) Wie kommst du schon auf 15x2? Ich kann auch nicht lesen, übersehen, dass die Ziel-Kupfermenge doppelt so groß ist. Dann sieht das richtig aus.



  • @wob

    @wob sagte in Zwei Aufgaben zu Mathe/Physik:

    Es kommt kein nnn in deiner Formel vor

    Das ist korrekt. Es gab zu a) noch andere Aufgaben, in denen ein n vorkam.

    @wob sagte in Zwei Aufgaben zu Mathe/Physik:

    Dann sieht das richtig aus.

    Das ist gut. Aber schlussendlich ist es ein Dreisatz... (oder?)

    Wie löse ich a)?

    Btw., das Ganze ist aus einem Buch, das ich "damals" zum lernen genutzt hatte.



  • @Fragender sagte in Zwei Aufgaben zu Mathe/Physik:

    Wie löse ich a)?

    In dem Du die Logarithmengesetze anwendest und anschließend die Gleichung exponenzierst.

    ln2x=2lna12lna3\ln \frac{2}{x}=2\ln a-\frac{1}{2}\ln\frac{a}{3}

    Das kann man vereinfachen in dem man log(ab)=loga+lnb\log (ab) = \log a +\ln b bzw. logab=logalogb\log\frac{a}{b}=\log a - \log b verwendet, und dann gibt es noch alogb=log(ba)a\log b=\log (b^a). Es sind weiterhin die folgenden Kurzformen üblich lna=logea\ln a = \log_e a, lga=log10a\lg a=\log_{10} a, d.h. der ln\ln wird zur Basis der Eulerschen Zahl berechnet. log\log steht für den Logarithmus zu einer nicht benannten Basis, die gibt man in der Form logc\log_c an, so dass hier zur Basis cc der Logarithmus zu berechnen wäre.



  • Cool, dass Formeln hier gehen!

    @john-0

    ln2x=2lna12lna3x=2ϵ1/2(3log(a)log(3))\ln \frac{2}{x}=2\ln a-\frac{1}{2}\ln\frac{a}{3} \Rightarrow x=2 \epsilon^{1/2 (-3 log(a) - log(3))}

    Ist das korrekt? Wie komme ich dahin? Hätte dafür nicht 5 Minuten, sondern mehrere Stunden gebraucht ...

    E: Das Epsilon soll die Eulersche Konstante sein.



  • @Fragender
    Also ich habe da raus: x=23a3/2x = \frac{2}{\sqrt{3}} a^{-3/2}



  • @Fragender sagte in Zwei Aufgaben zu Mathe/Physik:

    Cool, dass Formeln hier gehen!

    @john-0

    ln2x=2lna12lna3x=2ϵ1/2(3log(a)log(3))\ln \frac{2}{x}=2\ln a-\frac{1}{2}\ln\frac{a}{3} \Rightarrow x=2 \epsilon^{1/2 (-3 log(a) - log(3))}

    Ist das korrekt? Wie komme ich dahin?

    Ok, dann rechne ich das mal vor.

    ln2x=2lna12lna3\ln \frac{2}{x}=2\ln a-\frac{1}{2}\ln\frac{a}{3}

    den ersten Logarithmus auseinanderziehen

    ln2lnx=2lna12lna3\ln 2 -\ln x =2\ln a-\frac{1}{2}\ln\frac{a}{3}

    dann ln2\ln 2 auf die andere Seite bringen

    lnx=ln2+2lna12lna3-\ln x=-\ln 2+2\ln a-\frac{1}{2}\ln\frac{a}{3}

    Gleichung mit 1-1 multiplizieren

    lnx=ln22lna+12lna3\ln x = \ln 2-2\ln a+\frac{1}{2}\ln\frac{a}{3}

    die Faktoren vor den Logarithmen in dieselben ziehen

    lnx=ln2lna2+lna3\ln x = \ln 2- \ln a^2+\ln\sqrt{\frac{a}{3}}

    und dann die Logarithmen zusammenziehen

    lnx=ln2a2+lna3\ln x=\ln\frac{2}{a^2}+\ln\sqrt{\frac{a}{3}}

    lnx=ln(2aa23)\ln x=\ln\left(\frac{2\sqrt{a}}{a^2\sqrt{3}}\right)

    und als letztes wird die Gleichung potenziert

    x=2aa23x=\frac{2\sqrt{a}}{a^2\sqrt{3}}

    dann schreiben wir die Gleichung noch „schön“ in dem die Wurzel aus dem Nenner entfernen (mit 3\sqrt{3} erweitern)

    x=23a3a2=233a3/2x=\frac{2\sqrt{3a}}{3a^2}=\frac{2\sqrt{3}}{3}a^{-3/2}



  • Danke erstmal. Ich bin gerade auf dem Sprung, aber

    @wob ich glaube, Wolfram Alpha interpretiert ln als logarithmus naturalis ... kann das später mal überprüfen/nachrechnen.

    Bauchgefühlsmäßig kam mir das auch schon spanisch vor.



  • @Fragender sagte in Zwei Aufgaben zu Mathe/Physik:

    @wob ich glaube, Wolfram Alpha interpretiert ln als logarithmus naturalis ... kann das später mal überprüfen/nachrechnen.

    Was hat das mit Wolframalpha zu tun? ln ist der Logarithmus zur Basis e. Genau wie das in vielen Programmiersprachen (inkl. Wolframalpha) auch für log der Fall ist. Gerade getestet: Bei WA kannst du dir aussuchen, ob du ln oder log schreibst. Du bekommst bei log allerdings die Meldung Assuming "log" is the natural logarithm | Use the base 10 logarithm instead (wo du drauf klicken kannst und dann umstellen kannst)

    Es ist auch prinzipiell beim Rechnen egal, zu welcher Basis der log ist - ist eh nur ein konstanter Faktor. logax=lnx/lna\log_a x=\ln x / \ln a



  • @wob sagte in Zwei Aufgaben zu Mathe/Physik:

    Was hat das mit Wolframalpha zu tun?

    Na, weil als Ergebnis von WA etwas mit e rauskommt. Und WA wird sich ja nicht verrechnen.



  • Ist dein WA ein anderes? Natürlich kommt das raus, was @john-0 und ich geschrieben haben.

    https://www.wolframalpha.com/input?i=Solve[2+ln+a+-+1%2F2+ln(a%2F3+)%3D%3Dln(2%2Fx)%2C+x]





  • Oh shit ... Hast recht ... man sollte schon etwas mehr lesen, als nur eine Zeile: 😃

    https://i.postimg.cc/pdt0BPb2/grafik.png



  • @wob sagte in Zwei Aufgaben zu Mathe/Physik:

    Ist dein WA ein anderes? Natürlich kommt das raus, was @john-0 und ich geschrieben haben.

    https://www.wolframalpha.com/input?i=Solve[2+ln+a+-+1%2F2+ln(a%2F3+)%3D%3Dln(2%2Fx)%2C+x]

    @wob Die "Schritt-für-Schritt-Lösung" ist aber nur für Pro-Leute verfügbar.

    Ehrlich gesagt, hab ich es immer noch nicht ganz verstanden, obwohl es @john-0 ja schon aufgedröselt hatte.



  • @Fragender sagte in Zwei Aufgaben zu Mathe/Physik:

    Ehrlich gesagt, hab ich es immer noch nicht ganz verstanden, obwohl es @john-0 ja schon aufgedröselt hatte.

    An welcher Stelle hakt es?


Anmelden zum Antworten